Which one of the following, if substituted for the constraint that neither Xiao's speech nor Yoshida's speech can be ...

Seo on September 13, 2020

Rule Substitution

Hi! Can you please explain how to solve this substitution question? I went with the answer choice B and thought it would achieve the same effect as the rule, but why is this incorrect? How should I tackle substitution questions like this one? Thank you!

Reply
Create a free account to read and take part in forum discussions.

Already have an account? log in

Emil-Kunkin on May 4 at 10:48PM

Hi, B would allow a scenario in which X and Y are both at 1, M and Z are both at 3 and L is at 3. This is banned in the original rule. Generally for rule substitutions wrong answers are wrong for one of two reasons. First, the can allow some that that was previously banned, as was the case here. Second, they can ban something that was previously allowed.